Solved: Projection Theorem in Hilbert Space

In summary, the mathematician is trying to solve a problem in which two vectors must be equal in order for a third vector to be equal as well. If x-y is in a subspace M of a Hilbert space, then |x-y| must be less than |x-m| for all m in M, and so |x-y+z| must also be less than |x-m|. This is shown by the implication "<=", which states that if x-y is in M^{\perp} then x-y+z is also in M^{\perp}. However, the proof for "=>" is contained in the proof for "<=" and so is
  • #1
P3X-018
144
0
[SOLVED] Projection Theorem

Homework Statement


If M is a closed subspace of a Hilbert space H, let x be any element in H and y in M, then I have to show that

[tex] \|x-y\| =\inf_{m\in M}\|x-m\| [/tex]

implies (equivalent to) that

[tex] x-y\in M^{\perp} [/tex]


The Attempt at a Solution



I have shown the implication "<=", ie that [itex] x-y\in M^{\perp} [/itex] implies the 1st statement. And I've been told that the implication (=>) I now want to show is basically in the that proof.

The proof for "<=" goes like: If [itex] x-y\in M^{\perp} [/itex] then (x-y,z) = 0 for all z in M, so by Pythagoras thm

[tex] \|x-y+z\|^2 = \|x-y\|^2 +\|z\| \geq \|x-y\|^2 [/tex]

M subspace => [itex] m = y-z \in M[/itex]. So [itex] \|x-y\| \leq \|x-m\|[/itex] for all m in M. So this gives the implication the other way around.

But I don't see how I can go back, nor how the 'proof' for "=>" is basically contained in my this proof.
And how or where should I use that M is closed, I feel like it should have been used to conclude the implication "<=" from knowing [itex] \|x-y\| \leq \|x-m\|[/itex], or is it not needed for "<="?
 
Last edited:
Physics news on Phys.org
  • #2
Here's a summary of what you've already shown (assume x, y are fixed vectors, with y in M, and z is an arbitrary vector in M, so that w=z-y is also an arbitrary vector in M):

[tex]x-y \in M^\perp \Rightarrow (x-y,z)=0 \Rightarrow ||x-y+z||^2 = ||x-y||^2 + ||z||^2 \Rightarrow ||x-y|| \leq ||x-w|| \Rightarrow ||x-y|| = \inf_{m\in M}\|x-m\| [/tex]

Now to get the other direction, start by seeing how many of those [itex]\Rightarrow[/itex]'s you can turn into [itex]\Leftarrow[/itex]'s (or to get both directions at once, [itex]\Leftrightarrow[/itex]'s). The one that should give you the most difficulty is:

[tex] ||x-y+z||^2 = ||x-y||^2 + ||z||^2 \Leftarrow||x-y|| \leq ||x-y+z|| [/tex]

The condition can be written as:

[tex] ||x-y||^2 \leq ||x-y+z||^2 = ||x-y||^2 + 2 \Re (x-y,z) + ||z||^2 [/tex]

[tex] \Leftrightarrow 0 \leq ||z||^2 + 2\Re (x-y,z) [/tex]

Think about what happens when z is really small.
 
Last edited:
  • #3
So using Cauchy Schwartz, since (x-y,z) must be real, I get

[tex]0 \leq \|z\|^2 + 2(x-y,z) \leq \|z\|(\|z\| + 2\|x-y\|)[/tex]

So that [itex] \|z\|^2 + 2(x-y,z)[/itex] gets 'squeezed' to zero, and hence (x-y,z) too? But if z varies it'll affect (x-y,z) too, so that thing can get small too. So how does varying z helps me in this?
 
  • #4
Sorry, I'm not following your last post. One way to continue would be to write:

[tex]f(t) = ||tz||^2 + 2 \Re (x-y,tz) [/tex]

Given the above inequality, we can write:

[tex] 0 \leq f(t) = t^2||z||^2 + 2 \Re (x-y,z) t [/tex]

But's it's easy to see that this will be negative at some points unless [itex]\Re(x-y,z)=0[/itex]. There's a trick to get the same thing for the imaginary part. But I have a feeling there's a simpler way to do this.
 
Last edited:
  • #5
That was my mistake I thought (x-y,z) should be real so I used Cauchy-Schwartz, but never mind that.

I see that there could be some points where that inequality could fail, but doesn't that need to be proved first? t would have to fulfill

[tex] (t \|z\|^2 + 2Re(x-y,z))t < 0 [/tex]
[tex]0< t < -\frac{2Re(x-y,z)}{\|z\|^2}[/tex]

So that f(t) would be negative. Don't we need to show that such a z in M would exist? Ie any z such that [itex]-2Re(x-y,z)/\|z\|^2>0[/itex] or [itex]Re(x-y,z)<0[/itex].
Otherwise it is clear how Re(x-y,z) has to be 0.

But I'm a little clueless on how to show Im(x-y,z) = 0, what is the trick you were talking about?
 

Related to Solved: Projection Theorem in Hilbert Space

1. What is the Projection Theorem in Hilbert Space?

The Projection Theorem, also known as the Orthogonal Projection Theorem, is a fundamental result in functional analysis that states that any vector in a Hilbert Space can be uniquely decomposed into the sum of two orthogonal vectors, one of which lies in a closed subspace of the Hilbert Space.

2. How is the Projection Theorem used in mathematics and physics?

The Projection Theorem has various applications in mathematics and physics, particularly in the fields of functional analysis, linear algebra, and quantum mechanics. It is used to prove the existence of orthogonal bases and to derive the spectral theorem for self-adjoint operators, which has important implications in quantum mechanics.

3. Can you explain the concept of orthogonal vectors in the Projection Theorem?

Orthogonal vectors are vectors that are perpendicular to each other, meaning their dot product is equal to zero. In the context of the Projection Theorem, the orthogonal vectors refer to the two components of the decomposed vector, one of which lies in a closed subspace and the other in the orthogonal complement of that subspace.

4. Is the Projection Theorem applicable to any Hilbert Space?

Yes, the Projection Theorem is applicable to any Hilbert Space, which is a complete vector space equipped with an inner product. However, for the theorem to hold, the subspace must be closed, meaning it contains all its limit points.

5. Are there any generalizations of the Projection Theorem?

Yes, there are generalizations of the Projection Theorem, such as the Moore-Penrose pseudoinverse theorem and the generalized inverse theorem, which extend the concept of orthogonal projections to non-Hilbert spaces and non-orthogonal bases.

Similar threads

  • Calculus and Beyond Homework Help
Replies
3
Views
394
  • Calculus and Beyond Homework Help
Replies
3
Views
552
  • Calculus and Beyond Homework Help
Replies
8
Views
530
  • Calculus and Beyond Homework Help
Replies
8
Views
910
  • Calculus and Beyond Homework Help
2
Replies
43
Views
3K
  • Calculus and Beyond Homework Help
Replies
19
Views
1K
Replies
1
Views
658
  • Calculus and Beyond Homework Help
Replies
2
Views
1K
  • Calculus and Beyond Homework Help
Replies
10
Views
532
Replies
2
Views
1K
Back
Top